Henry gathered data about the types of nuts in five handfuls of mixed nuts. The data he gathered is shown in the table. Select the points that represent this data.

Henry Gathered Data About The Types Of Nuts In Five Handfuls Of Mixed Nuts. The Data He Gathered Is Shown
Henry Gathered Data About The Types Of Nuts In Five Handfuls Of Mixed Nuts. The Data He Gathered Is Shown

Answers

Answer 1

Answer:

Look below.

Step-by-step explanation:

Henry Gathered Data About The Types Of Nuts In Five Handfuls Of Mixed Nuts. The Data He Gathered Is Shown
Answer 2

The location of the coordinate plane will be shown in the graph.

What is coordinate geometry?

Coordinate geometry is the study of geometry using the points in space.

Henry gathered data about the types of nuts in five handfuls of mixed nuts.

The data he gathered is shown in the table.

Handful     Number of peanuts       Number of other nuts

  A                           9                                        7

  B                           6                                        5

  C                           8                                        9

  D                           5                                        7

  E                            7                                        4

The graph is shown below.

More about the coordinate geometry link is given below.

https://brainly.com/question/1601567

#SPJ2

Henry Gathered Data About The Types Of Nuts In Five Handfuls Of Mixed Nuts. The Data He Gathered Is Shown

Related Questions

On a unit circle, the terminal point of 0 is (square root of 2/2, square root of 2/2) what is 0?

Answers

Answer:

Step-by-step explanation:

0 should never be used as a substitute for θ!

(√2/2, √2/2) = (cos(45°), sin(45°))

The value of θ if 0°< 0<360° on a unit circle where the terminal point of 0 is (√2/2, √2/2), is 45°.

What is a terminal point?

The terminal point of a circle is defined as the measure of the angle in degrees or radians over the circle, at the endpoint on the coordinate plane.

On a unit circle, the terminal point of θ is,

(√2/2, √2/2) = (cos(45°), sin(45°))

The value of theta at the terminal point (square root of 2/2, square root of 2/2) is 45 degrees.

Thus, the value of θ if 0°< 0<360° on a unit circle where the terminal point of 0 is (square root of 2/2, square root of 2/2) is 45°.

Learn more about the terminal point here;

brainly.com/question/4256586

#SPJ2

please help me

Expand ( p + 6 )( p - 3 )​

Answers

Answer:

(p^2) + 3p - 18

Step-by-step explanation:

Have a nice day!

Answer:

2p+3

Step-by-step explanation:

(p+6)(p-3)

you have to open the brackets i.e

p+p+6-3

add p+p and you get 2p then you  subtract positive 6 from 3 and you get 3

so your answer will be 2p+3

Find the sum of 91.8+0.964+5.09 and correct your answer to one decimal place.

Answers

97.9
I used Socratic

Answer:

97.85

I hope this helps!

40) what is the area of a rectangular porch measuring 8 ft x 12/f

45) Create a stem and leaf plot to represent this set of data.
30, 62, 32, 63, 43, 77, 48, 78, 49, 82, 51, 84, 60,


please make sure to answer both questions ​

Answers

40. 8 times 12. = 96 ft

Please help its due in 30 minutes will mark braniliest

Answers

Answer:

Point A, B, and C are collinear points.

Step-by-step explanation:

When points lie on one line, meaning all of the points are on the same line --> this is known as collinear points.

please help. pls show workings ​

Answers

Answer:

≈ 10.52 cm²

Step-by-step explanation:

The unshaded area is calculated as area of square subtract area of quarter circle, thus

A = 7² - [tex]\frac{1}{4}[/tex]πr²

   = 49 - ( 0.25π × 7²)

   = 49 - (0.25π × 49)

   = 49 - 12.25π

   ≈ 10.52 cm² ( to 2 dec. places )

 

Answer:10.5cm^2

Step-by-step explanation:

Area of the square = L^2 =(7) ^2=49cm^2

Radius of quadrant = 7cm

Area of the quadrant =1/4 x πr^2

1/4 x 22/7 x (7) ^2

1/4 x 22/7 x 49

=38.5cm^2

Area of unshaded part= area of square - area of quadrant

49cm^2 - 38.5cm^2

=10.5cm ^2

Mears Taxi charges a $ 3.95 flat rate for a ride in the cab. In addition to that, they charge $ 0.75 per mile. Katie has no more than $ 16 to spend on a ride. At most, how many miles can Katie travel without exceeding her spending limit?

Answers

Answer:

About 16 miles

The equation:

.75x+3.95

At 16 miles Katie will have spent $15.95.

Answer:

16 miles

Step-by-step explanation:

3.95 + .75m < 16

Subtract 3.95 from each side

3.95 -3.95+ .75m < 16-3.95

.75m < 12.05

Divide each side by .75

.75m/.75  < 12.05/.75

m <16.066666666

Rounding to the nearest mile

m < 16

reduce to lowest term. 2x^2- 18/x^2+4x-21​

Answers

Answer:

[tex]\frac{2(x+3)}{x+7}[/tex]

Step-by-step explanation:

Given

[tex]\frac{2x^2-18}{x^2+4x-21}[/tex] ← factorise numerator and denominator

2x² - 18 ( factor out 2 from each term )

= 2(x² - 9) ← difference of 2 squares

= 2(x - 3)(x + 3)

and

x² + 4x - 21 = (x + 7)(x - 3) , then

= [tex]\frac{2(x-3)(x+3)}{(x+7)(x-3)}[/tex] ← cancel common factor (x - 3) on numerator/ denominator

= [tex]\frac{2(x+3)}{x+7}[/tex]

how many are 6 raised to 3 ???​

Answers

Answer:

216

Step-by-step explanation:

When you say "6 raised to 3", it means you multiply 6, 3 times. Meaning:

6✖️6✖️6=216

Hope this helped!

Have a nice day:)

help do y’all know this! Please if u do i need the answer

Answers

9.4 × [tex]10^{2}[/tex]

1) Move the decimal point 2 times to left in the number so that number, 940 so it's between 1-10: 9.4 or 9.40

2) After we moved our decimal point to the left, the decimal count would be 12 and that we moved 2 places to the left

3) We know that the notation is a x [tex]10^{b}[/tex]

a=9.4 and 6=2

9.4 × [tex]10^{2}[/tex]=a x [tex]10^{b}[/tex]

I hope I've helped!

Answer:

9.4x10^8

Step-by-step explanation:

940 million looks like

940,000,000. in scientific notation you would move the decimal left 8 spaces

prove sin^2 π/6 + cos^2 π/3 - tan^2 π/4 = -1/2​

Answers

=(

2

1

)

2

+(

2

1

)

2

−1

2

=

2

−1

Therefore, L.H.S = R.H.S

Choose the inequality that represents the following graph.

Answers

Answer:

It is c

Step-by-step explanation:

because five is bigger than zero

Answer:

[tex]\sf x > 0[/tex]

Step-by-step explanation:

The arrow points to all values greater than 0. And the point at 0 is not filled in. So, the inequality represents everything strictly greater than 0.

[tex]\bf x > 0[/tex]

___________________

Hope this helps!

Have a great day!

Help please!!! thx...

Answers

[tex]\\ \sf\longmapsto cos\theta=\dfrac{3}{\sqrt{13}}[/tex]

We know

[tex]\boxed{\sf sin^2\theta=1-cos^2\theta}[/tex]

[tex]\\ \sf\longmapsto \sin( \theta) = \sqrt{1 - { \cos}^{2} \theta } \\ \\ \sf\longmapsto \sqrt{1 - {( \frac{3}{ \sqrt{13} }) }^{2} } \\ \\ \sf\longmapsto \sqrt{1- \frac{ {3}^{2} }{ {( \sqrt{13)} }^{2} } } \\ \\ \sf\longmapsto \sqrt{1 - \frac{9}{13} } \\ \\ \sf\longmapsto \sqrt{\frac{13 - 9}{13} } \\ \\ \sf\longmapsto \sqrt{ \frac{4}{13} } \\ \\ \sf\longmapsto \frac{2}{ \sqrt{13} } [/tex]

type in symbols to make 3,7,12,2 equal 45

Answers

Answer:

The answer is (3×7) + (12×2) .

[tex](3 \times 7) + (12 \times 2)[/tex]

[tex] = 21 + 24[/tex]

[tex] = 45[/tex]

Which ordered pair would fall in the first quadrant of the coordinate plane?
A) (3, 10)
B) (0, 0)
C) (0, 10)
D) (3, 0)

Answers

Answer: A (3,10)

Source : i made it up

3 times the sum of a number and 4 is 20

Answers

Answer:

as per the statement we form an algebraic equation as

3(x+4) = 20

3x + 12 = 20

3x = 8

x = 8/3

as per my understanding that's the answer

HELP ASAP!!!! URGENT!!! LOOK AT SCREENSHOT! Identify all points and line segments in the picture below. This image has the potential for visual bias, so there is no alternative text. Select one: a. Points: A, B Line segments: bar(AB) b. Points: A, B, C, D Line segments: bar(AB) c. Points: A, B, C, D Line segments: bar(AB), bar(BC), bar(CD), bar(AD), bar(BD), bar(AC) d. Points: A, B, C, D Line segments: bar(AB), bar(AC), bar(BD)

Answers

Answer:

the answer is D

How many solutions does the system have?

Answers

Answer:

                B. no solutions

Step-by-step explanation:

Left sides of both equations are the same sum (8x+2y), so the right sides also has to be the same. They are not so  there is no solutions.

{If they are the same then system has infinitely many solutions.}

Pens cost 15 pence each. Rulers cost 20 pence each. Write down an expression for the cost of x pens and x rulers.

Answers

Answer:

C = 35x pence

Step-by-step explanation:

1 pen costs 15 , thus x will cost 15x

1 ruler costs 20, thus x will cost 20x

Total cost (C) will then be

C = 15x + 20x = 35x pence

The total cost of pens and rulers, C = 35x pence

What is Equation?

Equations are mathematical statements with two algebraic expressions flanking the equals (=) sign on either side.

It demonstrates the equality of the relationship between the expressions printed on the left and right sides. LHS = RHS is a common mathematical formula.

Coefficients, variables, operators, constants, terms, expressions, and the equal to sign are some of the components of an equation. The "=" sign and terms on both sides must always be present when writing an equation.

Given:

Cost 1 pens is 15.

Then, cost for x pen is 15x

Cost of 1 ruler is 20

Then, cost of x ruler is 20x

So, the total cost is

= 15x + 20x

= 35x

Learn more about Equation here:

https://brainly.com/question/29538993

#SPJ2

Algebra 1 need help ASAP

Answers

Answer:

It's A

Step-by-step explanation:

The money earned is 160 in 7 hours

The money earned is 100 in 3 hours

The slope = change in money / change in time

The slope = (160 - 100 ) / (7 - 3) = 60/4 = 15 dollars / hour

The slope means that for every hour he earns 15 dollars.

Find the value of w in the equation 3w + 15 = 60? Show
your work
. Based on your value of w, what is the value of the expression
below?
8w2 + 8w + 9
PLZ HELPPPP you’ll get all of my points ❤️

Answers

W=15

8(15)2+8(15)+9=369

Answer:

Part 1: Find w

w=15

Step-by-step explanation:

Part 1: Find w

Step 1:

3w+15=60. If this is true, then first: Subtract 15 from both sides.

Result: 3w=45

Step 2:

3w=45. If this is true, now: Divide by 3 to both sides.

Result/Answer: w=15.

Answer:

Part 2: If w=15, then what is 8w2+8w+9?

8w2+8w+9= Well I have 2 options. If you mean 8w^2+8w+9, then the answer is 14529. If you mean 16w+8w+9, then the answer is 369.

What were Malcolm's and Ravi's maximum speeds?

Answers

Answer:

Malcom's maximum speed = 200 km/h

Ravi's maximum speed = 320 km/h

Step-by-step explanation:

Represent the maximum speed of Malcolm and Ravi with equation as follows:

Let Malcom's speed be x, and Ravi's speed by y.

The average of speed is said to be 260 km/h. An equation to represent this is: [tex] \frac{x + y}{2} = 260 [/tex]

[tex] x + y = 260*2 [/tex]

[tex] x + y = 520 [/tex] => equation 1.

We are also told that when Malcom's speed (x) is doubled it equal 80 km/hr more than Ravi's speed (y). An equation can be created for this, which is [tex] 2x = y + 80 [/tex]

[tex] 2x - y = 80 [/tex] => equation 2

Now that we have 2 equations as a system, solve for the values of x and y simultaneously.

Add both equations together to eliminate y

[tex] x + y = 520 [/tex]

[tex] 2x - y = 80 [/tex]

[tex] 3x = 600 [/tex]

[tex] x = \frac{600}{3} [/tex]

[tex] x = 200 [/tex]

Plug in the value of x into equation 1 to find y.

[tex] 200 + y = 520 [/tex]

[tex] y = 520 - 200 [/tex]

[tex] y = 320 [/tex]

Malcom's maximum speed = x = 200 km/h

Ravi's maximum speed = y = 320 km/h

The perimeter of a rectangle is 40 inches. If the width is 9 inches, what is the area of the rectangle?

Answers

Answer:

99 in²

Step-by-step explanation:

Perimeter = 2(length) + 2(width)

40 = 2(length) + 2(9)

40 = 2L + 18

40 - 18 = 2l

22 = 2l

L = 11

Area = length x width

Area = 9 x 11 = 99

99 square inches

If my answer is incorrect, pls correct me!

If you like my answer and explanation, mark me as brainliest!

-Chetan K

Answer: 360

Step-by-step explanation:

you multiply 40 and 9.

first you multiply 9 and 0 that is 0 then 9 to 4 and that is 36 so you get 360

a shopkeeper sells a pant at 10% profit, if he would have sold it at rs.150 more than profit would have been 15% find the purchased price of the pant

Answers

Answer:

selling prices =10% 10+150+15%=180

It takes 250 \text{ g}250 g250, start text, space, g, end text of pasta and 240 \text{ ml}240 ml 240, start text, space, m, l, end text of sauce to make one batch of spaghetti, and it takes 600 \text{ g}600 g600, start text, space, g, end text of pasta and 700 \text{ ml}700 ml 700, start text, space, m, l, end text of sauce to make one batch of lasagna.

Answers

Answer:

250S + 600L > 4000

240S + 700L > 5000

Step-by-step explanation:

It takes 250 g of pasta and 240 ml of sauce to make one batch of spaghetti, and it takes 600g of pasta and 700ml of sauce to make one batch of lasagna.

Friedrich is a chef who wants to make spaghetti and lasagna using more than 4000 g of pasta and more than 5000 ml of sauce.

Let S denote the number of batches of spaghetti he makes and L the number of batches of lasagna he makes.

Write an inequality that represents the condition based on the number of grams of pasta.

Write an inequality that represents the condition based on the number of milliliters of sauce.

Scenario 1: Pasta

The chef wants to make spaghetti and lasagna using more than 4000 g of pasta.

S =the number of batches of spaghetti he makes

L =the number of batches of lasagna he makes.

We have, inequality based on the number of grams of pasta

250S + 600L > 4000

Scenario 2: Sauce

chef wants to make spaghetti and lasagna using more than 5000 ml of sauce.

S= the number of batches of spaghetti he makes

L= the number of batches of lasagna he makes.

We have, inequality based on the number of milliliters of sauce

240S + 700L > 5000

The two inequalities are:

250S + 600L > 4000

240S + 700L > 5000

Write your answer as a polynomial or a rational fraction in simplest form ​

Answers

Answer:

- 4x^2-15x

Step-by-step explanation:

(f-g)(x) = f(x) - g(x) = - 3x^2-8x-x^2-7x = - 4x^2-15x

WHERE ARE THE EXPERTS AND ACE!!!!!!! I NEED HELP PLS SHARE YO SMARTNESS!!!!! WILL GIVE BRAINLIEST AND RATE AND VOTE!!! EASY IM JUST NOT SMART

2 QUESTIONS

Answers

Hi, Laura! ;)

The answer to the first question is 5,62.

Here is the step by step solution:

They want to know the center of a distribution. Another word for center is middle. When you have a set of numbers, you can find the middle by calculating the mean, the "average" of the data set.

1) Add all the numbers: 12,8 + 5,7 + 7,9 + 1,3 + 3,2 + 2,8 = 33,7

2) Divide the sum by the quantity of numbers, which is 6:  33,7 / 6 = 5,616

The closest number to 5,616 is 5,62.

==========================================================

Now let's answer your second question.

The correct answer is, indeed, 0,14.

How do we get to this number? Here is the step by step solution:

1) First step is to find out how many possible outcomes you have. It is the

number of raffle tickets in the box: 200.

2) Then, let's define the number of tickets that are prizes: 4 + 8 + 16 = 28.

The probability of selecting a prize ticket is the number of prize tickets divided by all the raffle tickets in the box: 28 / 200 = 0,14.

Now, let's practice Probability with another Brainly problem:

https://brainly.com/question/16447117

Good luck, Laura! :)

Work out the mean for the data set below: 3, 5, 4, 3, 5, 6 Give your answer as a fraction. answer

Answers

Answer:

4 1/3

Step-by-step explanation:

3 + 5 + 4 + 3 + 5 + 6 = 26

26/6 = 4 2/6 (4 1/3)

Answer:

13/3

Step-by-step explanation:

To find the mean, add up all the numbers and divide by the number of terms

( 3+5+4+3+5+6) /6

26/6

Divide top and bottom by 2 to simplify the fraction

13/3

Does anyone know the answer

Answers

Answer:

It's False Because Hypotenuse will be the longest side of leg here

how many are 4 raised to 2 ???​

Answers

Answer:

[tex]\huge \boxed{16}[/tex]

Step-by-step explanation:

4 raised to 2 is 4 squared or 4 to the power of 2.

[tex]4^2 =4 \times 4 = 16[/tex]

Other Questions
convert the equation f(x)=1/2x^2+3x-2 to vertex form Which were included among the Three Principles of the People? Check all that apply. communism How would you write Twice the difference of 9 and a number. Why was Congregationalism important? A diamond ring was reduced from $999.99 to S789.99. Find the percent reduction in the price. Round the answer to the nearest tenth of a percent, ifnecessary.The reduction in price is? A uniform meter stick is hung at its center from a thin wire. It is twisted and oscillates with a period of 5 s. The meter stick is then sawed off to a length of 0.76 m, rebalanced at its center, and set into oscillation. With what period does it now oscillate? Please help. I need it. Bad. Just a young gun with a quick fuseI was uptight, wanna let looseI was dreaming of bigger things andWanna leave my own life behindNot a "Yes sir", not a followerFit the box, fit the moldHave a seat in the foyer, take a numberI was lightning before the thunderThunder, thunderThunder, thun-, thunderThun-thun-thunder, thunderThunder, thunder, thun-, thunderThun-thun-thunder, thunderThunder, feel the thunderLightning and the thunderThunder, feel the thunderLightning and the thunderThunder, thunderThunderKids were laughing in my classesWhile I was scheming for the massesWho do you think you are?Dreaming 'bout being a big starYou say you're basic, you say you're easyYou're always riding in the backseatNow I'm smiling from the stage whileYou were clapping in the nosebleedsThunder, thunder, thun-Thunder, thun-thun-thunderThunder, thunderThunder, thun-, thunderThun-thun-thunder, thunderThunder, feel the thunderLightning and the thunderThunder, feel the thunderLightning and the thunderThunderThunder, feel the thunderLightning and the thunderThunderThunder, feel the thunderLightning and the thunder, thunderThunder, feel the thunderLightning and the thunder, thunderThunder, feel the thunderLightning and the thunder, thunderThunder, feel the thunder (feel the)Lightning and the thunder, thunder Try this 33-33+33330 Marnie solved the proportion 150/170=x/510 to find the value of X Samm Corp. purchased a plot of land for $100,000. The cost to raze a building on the property amounted to $50,000 and Samm received $10,000 from the sale of scrap materials. Samm built a new plant on the site at a total cost of $800,000 including excavation costs of $30,000. What amount should Samm capitalize in its land account? a. $150,000. b. $140,000. c. $130,000. d. $100,000. A kitchen helper stacked some identical bowls into 2stacks.The height of the first stack of 6 bowls. is 16.82cm.The height of the second stack of 8 bowls in 21.2cm. A)Find the height of one bowl. 33. Find the area of the parallelogram above? n a genetics experiment on peas, one sample of offspring contained 372 green peas and 35 yellow peas. Based on those results, estimate the probability of getting an offspring pea that is green. Is the result reasonably close to the value of 3 4 that was expected? Which description correctly explains the deaths of Cassius and Brutus?Cassius stabs himself due to the information that Brutus has been killed; Brutus is killed by Octavius.Cassius and Brutus are both killed by Octavius.Cassius and Brutus are both killed by Mark Antony.Cassius asks Pindarus to kill him with his own sword; Brutus runs onto his own sword, which is held by Strato. what is the mass of cerrusite would contain 35g of lead? What is 64 divisible by? A 12-year capital lease specifies equal minimum annual lease payments. Part of this payment represents interest and part represents a reduction in the net lease liability. The portion of the minimum lease payment in Year 10 applicable to the reduction of the net lease liability should be: Tuition for one year at the University of Atlantis costs $12,000 per year. Rachel would like to attend this university and will save money each month for the next 3 years. Her parents will contribute $3,000 for her first year's tuition. How much money will Rachel need to save each month to have enough money for the first year of college at the University of Atlantis? Someone pls help me the answer choices are . increase , decrease, or remain the same